Тема Высшая проба
Высшая проба
Вспоминай формулы по каждой теме
Решай новые задачи каждый день
Вдумчиво разбирай решения
ШКОЛКОВО.
Готовиться с нами - ЛЕГКО!
Разделы подтемы Высшая проба
Подтемы раздела высшая проба
Решаем задачи

Ошибка.
Попробуйте повторить позже

Задача 1#80743

Сфера касается всех рёбер пирамиды, в основании которой лежит выпуклый 2024-угольник. Покрасим в шахматном порядке углы между последовательными рёбрами при вершине вне многоугольника в синий и красный цвета. Докажите, что произведение синусов половинок синих углов равно произведению синусов половинок красных.

Источники: Высшая проба - 2024, 11.6 (см. olymp.hse.ru)

Подсказки к задаче

Подсказка 1

Пусть в основании пирамиды лежит 2024-угольник А₁А₂…А₂₀₂₄, а О – её вершина. Для начала будет полезно определиться, что синих и красных углов будет равное количество, так как в пирамиде у нас чётное число боковых граней. Давайте рассмотрим одну боковую грань и попробуем найти синус нужного нам угла. Обратите внимание: нас просят найти синус от половинного угла грани. На какую формулу это может быть намеком?

Подсказка 2

Действительно, просто так искать синус половинного угла мы не очень умеем. Давайте рассмотрим квадрат синуса половинного угла и понизим его степень. sin²(α/2) = (1 – cos(α)) / 2. А вот cos(α) мы уже умеем находить, например, по теореме косинусов. Но мы всё ещё никак не использовали нашу сферу. Как может помочь то, что она касается всех ребер пирамиды?

Подсказка 3

Если сфера касается всех ребер, значит, в пересечении с нашей боковой гранью будет получаться окружность, вписанная в треугольную боковую грань пирамиды. Пускай такая окружность касается стороны OА₁ в точке B₁, стороны OА₂ - в точке B₂, а стороны А₁А₂ - в точке С₁. Тогда по теореме об отрезках касательных к окружности, проведенных из одной точки, А₁B₁ = А₁С₁ = x, А₂B₂ = А₂С₁ = y, OB₁ = OB₂ = z. Как тогда выражается sin²(α/2) через x, y, z?

Подсказка 4

Давайте воспользуемся теоремой косинусов для треугольника А₁А₂O, но выразим стороны через x, y, z. Чему тогда будет равно (1 – cos(α)) / 2?

Подсказка 5

После преобразований получаем sin²(α/2) = (А₁B₁* А₂B₂) / (OА₁ * OА₂), где α = ∠А₁OА₂. Заметьте, что данная формула будет верна, как для красных, так и для синих углов, если вместо 1 подставить i, а вместо 2 – (i + 1). Объясните, почему в таком случае произведение синих углов будет равно произведению красных.

Показать доказательство

Пусть в основании пирамиды лежит 2024-угольник A ...A
 1    2024  , точка O  — вершина пирамиды. Пусть для всех i∈ {1,2,...,2024} , сфера касается ребер OAi  в точке Bi  , а ребер AiAi+1  в точке Ci  (A2025 = A1  ).

Рассмотрим треугольник A1A2O  . Сечением сферы в его плоскости является вписанная в него окружность, которая касается его сторон в точках B1  , B2  , C1  . Пусть

A1B1 = A1C1 = x

A2B2 = A2C1 = y

OB1 = OB2 = z

∠A1OA2 =α

Из теоремы косинусов имеем

(x+ y)2 = (x+ z)2 +(z+ y)2− 2(x +z)(z +y)cosα

2xy = (x+ z)(y+ z)− (x+z)(z+y)cosα

1− cosα       xy
---2---= (x+z)(z+y)

1−-cosα-= A1B1-⋅ A2B2
   2     OA1   OA2

Как известно,

1−-cosα-    2 α
   2   = sin 2

По условию достаточно показать, что произведения квадратов половинных синих и половинных красных углов равны. Но из равенства выше каждое из таких произведений равно произведению отношений AiBi
OAi-  для всех i∈ {1,2,...,2024} , что доказывает исходное равенство.

Ошибка.
Попробуйте повторить позже

Задача 2#80742

Деревни некоторой языческой страны соединены дорогами так, что от любой деревни можно добраться до любой другой не проходя ни через какую деревню дважды, причём сделать это можно единственным способом. В каждой деревне живет свое племя туземцев. Каждое племя поклоняется одному из трёх идолов: Камню, Ножницам или Бумаге. Известно, что Камень сильнее Ножниц, Ножницы сильнее Бумаги, а Бумага сильнее Камня. Каждое племя желает, чтобы их идол был не слабее, чем идол любого из соседствующих с ними племен. С этой целью каждый вечер ровно в 20 :24  каждое племя смотрит на своих соседей и, если обнаруживает соседа с более сильным идолом, меняет свои верования, начиная поклоняться этому более сильному идолу. Верно ли, что рано или поздно все племена начнут верить в одного и того же идола?

Источники: Высшая проба - 2024, 11.5 (см. olymp.hse.ru)

Подсказки к задаче

Подсказка 1

Если видите в задаче ребят и дружеские связи, города, дороги и тому подобное, то сразу же переводите задачу на язык графов, так с ней намного удобнее будет работать. Обозначим города за вершины, а дороги между ними за ребра. Поверим в лучшее и попробуем доказать, что всё-таки все вершины будут иметь одного идола в конце. Подумайте над тем, какой перед нами граф и как он поможет нам в доказательстве.

Подсказка 2

В условии сказано, что существует единственный простой путь (без повторения вершин) между любыми двумя вершинами, значит, перед нами граф-дерево, а у такого графа есть «листья» (вершина, у которой есть только один «сосед»). Попробуйте доказать необходимое утверждение по индукции, рассмотрев граф на n вершин, для которого оно выполняется, а шагом индукции пусть будет добавление «листа».

Подсказка 3

Возьмем две вершины нашего графа, вершину A – лист, вершину B – родитель вершины A (единственный её «сосед»). Рассмотрите два случая, когда вершина B не меняла своего идола на идола вершины A и когда меняла. Если в каждом из случаев в графе идолы всех вершины становятся одинаковыми, то мы доказали утверждение.

Подсказка 4

Пусть B не меняла своего идола на идола вершины A, значит, либо идол B сильнее идола вершины A, либо их идолы равны. В каждом из этих случаев в один момент идолы вершин A и B будут равны, а по предположению индукции и весь граф будет иметь одинаковых идолов. Осталось аналогичным способом рассмотреть случай, когда B поменяла своего идола на идола вершины A.

Показать ответ и решение

Переформулируем условие задачи на язык графов. Каждому племени поставим в соответствие вершину, между любыми двумя соседними проведем ребро. Для каждой вершины определен ее идол, если идол соседней вершины сильнее, то в этот день вершина меняет своего идола на более сильного.

Будем решать задачу индукцией по количеству вершин. База индукции для графа, состоящего из одной вершины, очевидна.

Докажем, что если условие задачи верно для некоторого натурального n,  то оно же верно для n+ 1.  Выберем висячую вершину  A  исходного дерева, родителем которой является вершина B.

Будем говорить, что вершина X  влияет на вершину Y,  если данные вершины являются соседями и идол X  сильнее идола Y,  при этом Y  не имеет соседа с более сильным идолом, отличным от X.

Если не существует дня, в котором вершина A  влияло на вершину B.  В графе, полученном из данного удалением вершины A,  по предположению индукции все вершины через несколько дней будут иметь одного и того же идола, в частности его же будет иметь вершина B,  а значит и вершина A.

Предположим, что существует день, в котором A  повлияло на B.  После этого дня вершины A  и B  имеют одинакового идола. Теперь, если вершина B  меняет идола на более сильного, то на следующий день A  так же начинает поклоняться этому идолу, а значит не наступит дня, в котором A  имело бы более сильного идола, следовательно A  больше никогда не повлияет на B,  тем самым мы можем применить рассуждения, проделанные ранее, для дня, после которого вершина A  повлияла на B.

Ответ:

Да

Ошибка.
Попробуйте повторить позже

Задача 3#80741

Есть 4n  отрезков длины x
 1  , x
 2  , …, x
 4n  , где x = 1
 1  , x  =2
 2  , а при k> 2  выполнено x = x  + x
k   k−1   k−2  . Сколькими способами эти отрезки можно разбить на четвёрки так, чтобы из отрезков каждой четвёрки можно было составить четырёхугольник?

Источники: Высшая проба - 2024, 11.4 (см. olymp.hse.ru)

Подсказки к задаче

Подсказка 1

Подумайте, при каком условии из четырех отрезков можно составить четырехугольник. Вспомните аналогичное условие для треугольников.

Подсказка 2

Да, из отрезком a < b < c < d можно составить четырёхугольник <=> a+b+c > d, попробуйте рассмотреть произвольную четвёрку, которая образует четырёхугольник и расписать свойство x_k = x_{k-1} + x_{k-2} более подробно, т.е. для x_{k-1} применить его же и посмотреть чему тогда должно быть равно c и b.

Подсказка 3

Верно, если d = x_{k}, то b = x_{k-2}, c = x_{k-1}, иначе четырёхугольник не построится. Теперь задача свелась к подсчету количества способов выбрать n пар из тройки элементов и одного элемента, который не превосходит по номеру элементы выбранной тройки.

Подсказка 4

Введем понятие "хорошей" последовательности, состоящей из 2n чисел, в которой каждое из чисел 1, ..., n участвует ровно два раза. Как мы можем восстановить способ разбиения последовательности отрезков по хорошей последовательности? Может мы можем первому вхождению числа в "хорошую" последовательность сопоставить число, а второму - тройку?

Подсказка 5

Теперь давайте подсчитаем количество хороших последовательностей. Сколькими способами можно выбрать индексы для двух единиц? А сколько тогда останется возможных индексов для двух двоек? А сколько всего получится способов сопоставить каждому числу 2 индекса?

Подсказка 6

А не посчитали ли мы что-либо несколько раз? Меняет ли перестановка чисел в "хорошей" последовательности набор отрезков?

Показать ответ и решение

Из отрезков a< b< c< d  можно сложить четырехугольник тогда и только тогда, когда a+ b+ c> d  . Рассмотрим четверку xs < xi < xj < xk  , заметим, что xk =xk−1+ xk−2 = 2xk−2 +xk−3 > xk−2+xk−3+ xk−4  , следовательно, xj = xk−1  , иначе проверяемое неравенство не выполнено. Аналогично, можно показать, что xi =xk−2  .

Назовем последовательность     4n
{xi}i=1  интересной. Таким образом, необходимо посчитать количество способов выбрать в интересной последовательности n  пар из тройки элементов и одного элемента, который не превосходит по номеру элементы выбранной тройки.

Рассмотрим последовательность, состоящую из 2n  чисел, в котором каждое из чисел 1,...,n  участвуют ровно два раза и назовем ее хорошей. Восстановим по хорошей последовательности способ разбиения интересной последовательности. На первом шаге рассмотрим первое число в каждой из последовательности. На каждом следующем шаге, если рассматриваемое число в хорошей последовательности встречается впервые, то ставим ему в соответствие рассматриваемое число в интересной последовательно, после чего рассматриваем следующий числа в каждой из последовательностей. Если рассматриваемое число в хорошей последовательности встречается во второй раз, то ставим ему в соответствие тройку из рассматриваемого элемента в интересной последовательности и двух элементов, идущих после него. Таким образом, к концу процесса, каждому первому вхождению числа в хорошей последовательности стоит в соответствие один элемент интересной последовательности, а каждому второму тройка подряд идущих элементов интересной последовательности.

Посчитаем количество хороших последовательностей. Существует C22n  способов выбрать индексы двум единичкам, после этого останется 2n− 2  возможных индекса, следовательно, существует ровно C22n−2  способов выбрать индексы для двух двоек. Продолжая ставить каждому из чисел в соответствие два индекса, получим что общее количество способов сделать это, равно (2n)!∕2n  . Осталось заметить, что каждая перестановка чисел в хорошей последовательности не меняет набор разбиение интересной последовательности, следовательно, каждое разбиение было посчитано n!  раз (количество перестановок длины n  ), а значит общее количество разбиений равно

(2n)!= --(2n)!---= (2n− 1)!!
2nn!  2⋅4⋅...⋅2n
Ответ:

 (2n− 1)!!

Ошибка.
Попробуйте повторить позже

Задача 4#80740

В выпуклом четырёхугольнике ABCD  выполнено AB = BC = CD  . Его диагонали AC  и BD  пересекаются в точке E  . Описанная окружность треугольника ADE  пересекает сторону AB  в точке P ⁄= A  и продолжение стороны CD  в точке Q ⁄= D  . Докажите, что отрезки AP  и DQ  равны.

Источники: Высшая проба - 2024, 11.3 (см. olymp.hse.ru)

Подсказки к задаче

Подсказка 1

Заметьте, что AP и QD – хорды окружности, описанной около треугольника AED. Значит, чтобы доказать, что они равны, нам требуется доказать, что на данные хорды опираются равные вписанные углы. То есть если мы докажем, что углы QAD и ADP равны, то решим задачу. Подумайте, при каком условии данные углы могут быть равны.

Подсказка 2

Обратите внимание, что углы QAD и ADP – накрест лежащие для прямых PD и AQ, а значит, если мы докажем параллельность данных прямых, то решим задачу.

Подсказка 3

В условии не просто так нам дали, что три стороны четырехугольника попарно равны. Давайте рассмотрим равнобедренные треугольники ABC и BCD, а конкретно, рассмотрим их равные углы при основаниях. Подумайте, как они могут помочь в доказательстве параллельности прямых PD и AQ.

Подсказка 4

Рассмотрим два соответственных угла AQD и PDC. Из вписанности четырехугольника AQDE следует равенство ∠AQD = ∠DEC. Обратите внимание, что DEC является внешним углом треугольника BCE, значит, он равен сумме углов EBC и ECB. Вспомним про равнобедренные треугольники: в них есть два равных угла ∠EBC = ∠BDC. Значит, для решения задачи остается доказать, что ∠PDB = ∠ECB. Подумайте, как в этом может помочь окружность.

Подсказка 5

Четырехугольник APDQ является вписанным, значит, углы PAE и PDE будут равными, а угол PAE будет равен углу BCA, так как это углы при основании равнобедренного треугольника.

Показать доказательство

Первое решение.

Из вписанности четырехугольника APED  следует, ∠P AE = ∠EDP  . Треугольник ABC  является равнобедренным, а значит ∠BAC  =∠BCA  , следовательно, ∠BCA = ∠BDP  .

Из равнобедренности треугольника следует, что ∠DBC = ∠CDB  .

PIC

Наконец, в силу вписанности четырехугольника AEDQ

∠PDC = ∠PDB + ∠BDC = ∠BCE + ∠DBC = ∠CED  =∠AQD

Второе решение.

Достаточно показать, что хорды AP  и QD  стягивают равные дуги в окружности (AED )  , то есть доказать равенство ∠P EA =∠QED  . По теореме о внешнем угле верно, ∠P EA =∠BP E − ∠P AE.

Поскольку треугольник ABC  является равнобедренным ∠BAE = ∠BCE  , а из вписанности четырехугольника AP ED  следует, что ∠BP E =∠EDA  . Таким образом,

∠PEA = ∠BP E− ∠PAE = ∠EDA − ∠BCE

Аналогично,

∠DEQ = ∠DAE − ∠EBC

PIC

Наконец, исходное равенство углов можно переписать в виде

∠EDA − ∠BCE = ∠DAE − ∠EBC

∠EDA  +∠EBC  =∠DAE  +∠BCE,

что верно, так как суммой углов в каждой части равна углу между диагоналями четырехугольника.

Ошибка.
Попробуйте повторить позже

Задача 5#80739

В некотором числе 10 единиц, 100 двоек, 1000 троек, …, 109  девяток, расположенных в некотором порядке. Каждую секунду в нём стирают последнюю цифру. Правда ли, что в какой-то момент после начального получится число, делящееся на 9?

Источники: Высшая проба - 2024, 11.2 (см. olymp.hse.ru)

Подсказки к задаче

Подсказка 1

Давайте подумаем, каким образом нам можно число, которое кратно 9, независимо от остатка, который будет нами получен на каждом этапе вычеркивания. Удобная конструкция для нас - чтобы в течение 9 шагов у нас постоянно менялся остаток и не повторялся. Тогда, за 9 шагов у нас точно будет момент, когда остаток равнялся 0. Попробуйте придумать такую конструкцию.

Подсказка 2

Давайте попробуем вычеркнуть все 9 из числа(действительно, к чему бы они, если на деление на 9 они никак не влияют). Значит, если докажем, что в какой-то момент было число кратное 9 у полученного числа, то и у начального оно тоже было. Также, заметим, что под нашу конструкцию из первой подсказки подходит вариант, когда у нас стоит много одинаковых цифр подряд(хотя бы 9), взаимнопростых с 9, ведь там будет постоянно меняться остаток. То есть, нам надо набрать много одинаковых цифр подряд. Как это можно сделать?

Подсказка 3

Заметим, что чисел 8 у нас очень много. Больше чем 9 раз суммарно всех остальных. Давайте разобьем наше число на блоки по 9 цифр, которые не пересекаются. Что можно сказать про эти блоки? А что тогда надо доказывать в условиях на восьмерку?

Подсказка 4

Остается доказать, что найдется блок из цифр, равных 8. И это правда, так как иначе, в каждом блоке есть цифра, которая не 8 и тогда, цифр, не равных 8, у нас хотя бы 1/9 от общего количества. Противоречие. Значит, есть блок восьмерок. Победа.

Показать ответ и решение

Заметим, что если для исходного числа существует такой момент, то и для числа A  , полученного вычеркиванием всех девяток из исходного, он так же существует, поскольку каждое вычеркивание не меняет остаток при делении суммы цифр на 9.

Рассмотрим число A  . В силу неравенства   8      7
10 > 9⋅(10 + ...+ 10)  , отношение количества восьмерок к оставшимся числам, больше 9. Отметим подряд идущие блоки по 9 чисел. Докажем, что существует блок, элементами которого являются лишь восьмерки. Пусть это не так, тогда в каждом блоке есть цифра отличная от восьмерки, следовательно, количество цифр, не являющихся восьмерками, хотя бы   1∕9  от общего количество, что противоречит полученному неравенству.

Рассмотрим блок, состоящий только из восьмерок. Пусть число, полученное из A  вычеркиванием всех цифр до найденного блока, имеет остаток s <9  при делении на 9. Каждое вычеркивание 8 увеличивает остаток при делении на 9 на 1, следовательно, вычеркнув 9− s  элементов в блоке, мы получим искомое число.

Ответ: да

Ошибка.
Попробуйте повторить позже

Задача 6#80738

Можно ли число 2024 представить в виде a5+b3  , где a  и b  — натуральные числа?

Источники: Высшая проба - 2024, 11.1 (см. olymp.hse.ru)

Подсказки к задаче

Подсказка 1

Если a ≥ 5, то левая часть уже слишком большая. Остаётся перебрать четыре значения для а и проверить, чему равно b

Подсказка 2

Должен найтись подходящий вариант!

Показать ответ и решение

                  3  10   5   3
2024 =1000+ 1024= 10 +2  = 4 +10
Ответ: да

Ошибка.
Попробуйте повторить позже

Задача 7#75129

Однажды 45  друзей, живущих в разных уголках земного шара, захотели обменяться друг с другом новостями. Для этого они собираются устроить k  видеовстреч, на каждой из которых каждый человек расскажет всем свои новости, а также все новости других людей, которые он узнал ранее.

Для видеовстреч было предложено 10  дней, но оказалось, что каждый из друзей может присутствовать только в какие-то 8  из них. При каком наименьшем натуральном k  можно гарантированно выбрать k  дней для видеовстреч из предложенных 10  так, чтобы каждый узнал новости каждого?

(Между предложенными днями у людей новых новостей не возникает, и никак иначе они друг с другом не общаются. В каждый из предложенных дней проходит одна видеовстреча, на которой собираются все, кто может в этот день присутствовать.)

Показать ответ и решение

Оценка. Приведём пример ситуации, в которой 4 дней не хватит. Пусть у каждого из 45 людей будет своя, не совпадающая с другими людьми, пара дней, в которые он не может участвовать во встрече. Так как количество способов выбрать пару дней из 10 предложенных равно  2
C10 =45  , то для любой пары дней найдётся человек, который не может присутствовать ровно в эту пару дней. Предположим, что мы смогли выбрать какие-то 4 дня так, чтобы каждый узнал все новости. Но тогда существует человек A  , который не может присутствовать в первые два дня из этих четырёх, а также человек B  , который не может присутствовать в последние два из этих четырёх дней. Заметим, что тогда B  не сможет узнать новостей A  . Противоречие.

Пример. Теперь поймём, что 5 дней всегда точно хватит. Выберем 5 дней произвольным образом. Докажем, что любые два человека будут вместе присутствовать на какой-то встрече. Действительно, среди этих 5 дней есть не более 2 дней, в которые не может присутствовать первый, а также не более 2 дней, в которые не может присутствовать второй. Значит, найдётся день, в который могут присутствовать оба человека. Таким образом, каждая пара людей сможет обменяться новостями, то есть каждый узнает новость каждого.

Ответ:

При k= 5

Ошибка.
Попробуйте повторить позже

Задача 8#67770

Дана клетчатая доска 100× 100.  Каждая клетка доски покрашена в один из двух цветов: белый или чёрный. Назовём раскраску доски уравновешенной, если в каждой строке и в каждом столбце 50 белых и 50 чёрных клеток. За одну операцию разрешается выбрать две строки и два столбца так, чтобы из 4 клеток на их пересечении две были чёрными, а две — белыми, и перекрасить каждую из этих 4 клеток в противоположный цвет. Докажите, что из любой уравновешенной раскраски можно получить любую другую уравновешенную раскраску с помощью указанных операций.

Источники: Высшая проба - 2023, 11.5 (см. olymp.hse.ru)

Подсказки к задаче

Подсказка 1

Непонятно, как доказывать, что возможно перевести любую уравновешенную раскраску в любую другую. Может, тогда докажем, что любую уравновешенную раскраску можно перевести в какую-то конкретную, тогда из любой мы будем приходить в неё и уходить в любую другую, совершая в обратном порядке действия, с помощью которых мы бы приходили к ней. Надо подумать, к какой раскраске мы будем приходить...

Подсказка 2

На ум приходит, конечно же, шахматная раскраска. Но переводить доску целиком в шахматную как-то тяжеловато. Может, начать со столбцов?

Подсказка 3

Разобьем наши столбцы на пары и будем по очереди красить их в шахматную раскраску. Возьмем сейчас самую левую пару столбцов, которая еще не покрашена нужным нам образом. Будем приводить горизонтальные доминошки к шахматной раскраске сверху вниз. Что делать, если в какой-то момент у нас доминошка, которая содержит оба цвета, неправильно стоит?

Подсказка 4

Пускай для определённости левая клетка нашей доминошки черная:

Подсказка 5

А вывод такой: в какой-то доминошке, которая находится ниже нашей, левая клетка будет белой, а правая чёрной, ведь иначе суммарно чёрных клеток будет больше в первом столбце, чем во втором, а это противоречит тому, что раскраска доски уравновешенная. Тогда мы можем применить к этим двум доминошкам операцию и продолжить спуск вниз. Но что же делать, если в какой-то момент, идя вниз по столбцам, мы найдём одноцветную доминошку?

Подсказка 6

Пускай она чёрная. Можно заметить, что тогда ниже нашей доминошки существует белая доминошка (иначе в сумме по этим столбцам чёрных клеток будет слишком много). Что мы можем тогда сказать про строки, которые содержат наши доминошки?

Подсказка 7

На самом деле эти строки содержат две клетки, которые находятся в одном столбце, который находится правее наших, и при этом верхняя будет белая, а нижняя черная (Назовем этот столбец S). Это верно в силу того, что левее наших столбцов в этих строках поровну черных и белых клеток. Тогда мы можем выбрать один из наших столбцов, столбец S и поменять цвет клеток в них по этим строкам. Как же завершить решение?

Подсказка 8

Можно заметить, что, когда мы совершали операции по смене цвета, мы не нарушали уравновешенность таблицы. А это значит, что можно продолжать раскрашивать пары столбцов дальше и прийти к шахматной раскраске.

Показать доказательство

Докажем, что из любой уравновешенной доски можно получить доску, раскрашенную в шахматную раскраску, причём на каждом шаге доска будет оставаться уравновешенной. Из этого будет следовать, что из любой уравновешенной доски можно получить любую другую, так как операция обратима.

Будем получать шахматную раскраску следующим образом. Разобьём столбцы на пары подряд идущих. Выберем самую левую пару столбцов и в этой паре столбцов по очереди будем приводить строки (состоящие из двух клеток) к шахматной раскраске. После того как закончим с первой парой столбцов, перейдём ко второй и так далее.

Объясним, как делать следующий шаг внутри одной пары столбцов X  и Y.  Пусть в следующей строке A  сейчас находятся чёрная и белая клетка, но в неправильном порядке. Например, слева стоит чёрная клетка, а справа белая, а должно быть наоборот. Заметим, что во всех строках выше A  в первом столбце суммарно чёрных клеток не меньше, чем во втором, так как они уже покрашены шахматным образом. Значит, в какой-то строке B  ниже A  должна быть ситуация, когда в левом столбце чёрных клеток меньше, чем в правом, т.е. должна быть строка белая-чёрная (это следует из того, что суммарно в первом столбце столько же чёрных клеток, сколько и во втором). Произведём операцию со строками A  и B  и текущими столбцами.

PIC

Пусть теперь у нас в строке A  стоят две одинаковые клетки, например чёрные. Тогда в какой-то строке B  должны оказаться две белые клетки (иначе суммарно чёрных клеток в этих двух столбцах будет слишком много). Понятно, что эта строка расположена ниже текущей, т.к. выше неё все строки разноцветные. Теперь заметим, что если посмотреть на эту пару строк во всей таблице, то должен быть столбец Z  правее X  и Y,  в котором в первой строке белая клетка, а во второй — чёрная. Тут мы пользуемся тем, что левее наших столбцов в этих строках поровну чёрных и белых клеток. Теперь осталось лишь выбрать один из столбцов X  или Y  (в котором неправильный цвет в строке A)  и столбец Z,  а также строки A  и B  и произвести операцию с ними.

PIC

Легко видеть, что на каждом шаге уравновешенность доски сохраняется. А так как мы всегда можем сделать шаг в нашем алгоритме, то в конце получится шахматная раскраска.

Ошибка.
Попробуйте повторить позже

Задача 9#67769

В окружность ω  вписан треугольник ABC  такой, что AB < BC.  Биссектриса внешнего угла B  пересекает ω  в точке M.  Прямая, параллельная BM,  пересекает стороны BC,  AB  и продолжение стороны CA  за точку A  в точках P,Q  и R  соответственно. Прямая MR  вторично пересекает ω  в точке X.  Докажите, что точки B,P,Q,X  лежат на одной окружности.

Источники: Высшая проба - 2023, 11.4 (см. olymp.hse.ru)

Подсказки к задаче

Подсказка 1

Нам надо как-то воспользоваться тем, что BM паралельна PR. Например, можно перекинуть уголочек BMX на XRP как накрест лежащий. Куда ещё его можно перекинуть?

Подсказка 2

Т.к. BMX и BCX опираются на дугу BX мы получаем, что BMX=BCX. Не видно ли на картинке ещё одного вписанного четырехугольника?

Подсказка 3

Посмотрим на четырехугольник RXPC: XRP=XRQ=BMX=BCX=PCX. Тогда XRP=PCX, откуда следует, что RXPC вписан в окружность. Надо попробовать поперекидывать уголки в нем...

Подсказка 4

Нам необходимо доказать, что BPQX- вписан. Через какое равенство углов нам удобнее всего это сделать, если мы уже видим две окружности?

Подсказка 5

Наверное, через углы XBQ и XPQ, т.к. XBQ=XBA, а XPQ=XPR. Попробуйте перекинуть XBA на описанной окружности треугольника ABC, а уголок XPR на описанной окружности четырехугольника RXPC и вы завершите решение

Показать доказательство

PIC

Докажем, что точки R,X,P,C  лежат на одной окружности Ω,  т.е. что четырёхугольник RXP C  является вписанным. Действительно, ∠XRP  =∠BMX  как накрест лежащие при параллельных прямых BM  и RP  и секущей RM,  а ∠BMX  = ∠BCX  как опирающиеся на одну дугу в ω,  значит, ∠XRP  = ∠XCP.  Следовательно, по признаку четырёхугольник RXP C  является вписанным.

Из этого получаем, что ∠XCA  =∠XP R.  Из окружности ω  получаем, что ∠XBQ  = ∠XCA.  Значит, ∠XBQ = ∠XP Q,  а, следовательно, по признаку четырёхугольник XBP Q  является вписанным, т.е. точки X,B,P,Q  лежат на одной окружности.

Ошибка.
Попробуйте повторить позже

Задача 10#67768

Натуральные числа a,b,c  таковы, что 1≤ a< b< c≤ 3000.  Найдите наибольшее возможное значение величины

НОД(a,b)+ НОД (b,c)+ НОД (c,a)

Источники: Высшая проба - 2023, 11.3 (см. olymp.hse.ru)

Подсказки к задаче

Подсказка 1

Хочется получить какую-то оценку на эти НОДы. Понятно, что НОД(а,b) не больше чем a. Может попробовать что-то поделать с такой оценкой?

Подсказка 2

Если оценить так каждый НОД, получается как-то слишком грубо. Какой алгоритм приходит в голову, когда речь идёт о НОДах? Конечно же, алгоритм Евклида! Может, как-то улучшить оценку для НОД(a,b) с помощью него?

Подсказка 3

Если воспользоваться алгоритмом Евклида получиться, что: НОД(a,b)=НОД(a,b-a). Тогда т.к. НОД(a,b-a) не больше чем b-a, то и НОД(a,b) будет не больше чем b-a. если сделать тоже самое с НОД(b,c), что получится?

Подсказка 4

Итак, мы имеем что НОД(a,b) не больше b-a, а НОД(b,c) не больше c-b. Если их сложить, получится, что их сумма не больше чем (b-a)+(c-b)=с-а. Если оценить, что НОД(а,с) не больше чем a, то окончательно сумма всех НОДов будет не больше с, которое не больше 3000. Осталось только придумать пример...

Подсказка 5

Понятно, что с=3000. Чтобы достигалась оценка, необходимо, чтобы НОД(a,c)=a. Тогда с делится на a. Если мы сделаем так, что b тоже поделится на a, то, возможно, придумаем пример

Показать ответ и решение

Воспользуемся алгоритмом Евклида для Н ОД(a,b),  получим

НОД(a,b)= НО Д(a,b− a)

Заметим, что, так как НОД двух натуральных чисел не превосходит каждое из них,

НОД (a,b− a)≤ b− a⇒ НО Д(a,b)≤b − a

Аналогично получаем, что

НО Д(b,c)≤c− b

А также

Н ОД(c,a)≤ a

Складывая эти три неравенства, получаем

Н ОД(a,b)+ НОД(b,c)+ НОД(c,a)≤ (b− a)+ (c− b)+a = c≤3000

В качестве примера на 3000  можно предъявить, например, a= 1000,b= 2000,c= 3000.  В этом случае

НО Д(a,b)+Н ОД(b,c)+Н ОД(c,a)= 1000+ 1000 +1000= 3000
Ответ: 3000

Ошибка.
Попробуйте повторить позже

Задача 11#67767

Различные действительные числа x,y,z  таковы, что среди трёх чисел

---x+-y---  ---y+-z--   --z-+x----
x2+ xy+ y2 , y2+ yz+z2,  z2+zx+ x2

какие-то два равны. Верно ли, что все эти три числа равны?

Источники: УТЮМ - 2016 и Высшая проба - 2023, 11.2

Подсказки к задаче

Подсказка 1

Эти знаменатели подозрительно напоминают разложение разности кубов... Может, у каждой дроби умножить числитель и знаменатель на что-то и получить заветную разность?

Подсказка 2

Так и сделаем: числитель и знаменатель первой дроби умножим на x-y, второй на y-z, третьей на z-x и получим в числителях разность квадратов, а в знаменателях разность кубов. Но кажется, что это нам пока не сильно помогло...

Подсказка 3

Если уж какие-то два числа равны, то давайте приравняем первую и третью дроби (не умоляя общности) и посмотрим, что получится. (Похоже, что без работы ручками нам не обойтись...)

Подсказка 4

Перемножив крест-накрест и раскрыв скобки мы видим какой-то ужас. Хотя, если приглядеться, полученное равенство будет симметрично относительно переменных x и y. На какую мысль это наводит?

Подсказка 5

А мысль то проста: произвести все операции в обратном порядке, поменяв при этом местами переменные x и y, и получить равенство второй и третьей дроби!

Показать ответ и решение

В данных выражениях умножим числители и знаменатели на x− y,y− z,  z− x  соответственно (согласно условию, эти разности ненулевые). Получим те же числа в другом виде:

x2− y2  y2− z2   z2− x2
x3− y3, y3−-z3,  z3− x3

Без ограничения общности будем считать, что первое и третье числа равны. Тогда

 2  2    2  2
x3−-y3-= z3− x3-⇔
x − y   z − x

x2z3− x5− y2z3+ y2x3 = z2x3− z2y3− x5+ x2y3 ⇔

x3y2+ y3z2+ z3x2 = x3z2+ y3x2 +z3y2

Это симметричное равенство, поэтому теперь можно просто поменять местами две переменные (например, x  и y)  и проделать те же переходы в обратном порядке, получив равенство третьего и второго чисел:

x3y2+ y3z2+ z3x2 = x3z2+ y3x2 +z3y2 ⇔

x2z3 − x2y3− z5+ z2y3 = z2x3− z5− y2x3+ y2z3 ⇔

x2− z2   z2− y2
x3−-z3-= z3− y3-⇔

 2  2    2  2
z3− x3-= y3− z3
z − x   y − z

Деления при этом корректны, так как выражения-делители уже фигурировали ранее в знаменателях, и мы знаем, что они не равны нулю.

Ответ: верно

Ошибка.
Попробуйте повторить позже

Задача 12#67766

Каждое натуральное число покрасили в один из трёх цветов: красный, синий или зелёный, причём все 3 цвета встречаются. Может ли оказаться так, что сумма любых двух чисел разных цветов является числом оставшегося цвета?

Источники: Высшая проба - 2023, 11.1 (см. olymp.hse.ru)

Подсказки к задаче

Подсказка 1

Если ответ да, то как доказать, что такое возможно? Привести пример раскраски... Вроде как сходу такую раскраску придумать не получается. Может тогда воспользоваться методом от противного...

Подсказка 2

Пускай такая раскраска существует. Разумно было бы посмотреть на подряд идущие числа: ведь если они разного цвета, то их разность обязана быть покрашена в оставшийся цвет. А их разность это всегда 1

Подсказка 3

Возьмем числа 1 и n такие, что их цвета не совпадают. Тогда числа 1, n и n+1 покрашены в три различных цвета. Может попробовать пойти дальше и посмотреть на n+2? Какой же цвет имеет n+2=1+(n+1)? А n+3=1+(n+2)?

Подсказка 4

Получается, что n+2 имеет цвет числа n, а n+3 имеет цвет числа n+1. Похоже, что мы больше никогда не увидим число, цвет которого совпадает с цветом числа 1:( А может ли быть такое?

Подсказка 5

Такого, конечно же, не может быть: достаточно просто посмотреть на цвет числа 2n+1=n+(n+1)

Показать ответ и решение

Пойдём от противного, предположим, что такое возможно. Без ограничения общности можно считать, что число 1 покрашено в красный. Выберем произвольное число x,  покрашенное в синий. Заметим, что тогда x +1  должно быть зелёного цвета, (x +1)+ 1= x+ 2  — синего, (x+ 2)+ 1= x+ 3  — зелёного и т.д. Таким образом, все числа, большие x,  покрашены в синий или зелёный цвет. С другой стороны, так как x  покрашен в синий цвет, a x +1  — в зелёный, то число (x+ 1)+x =2x+ 1  должно быть покрашено в красный цвет, противоречие. Значит, такое невозможно.

Ответ: нет

Ошибка.
Попробуйте повторить позже

Задача 13#74783

Фокусник и его Ассистент готовятся показать следующий фокус. Фокуснику завяжут глаза, после чего один из зрителей напишет на доске 60-битное слово (последовательность из 60 нулей и единиц). Ассистент уверен, что сможет незаметно сообщить фокуснику 44 бита (не обязательно написанные в слове, можно вычислять любые функции). После чего Фокусник должен будет назвать слово. Для какого наибольшего числа C  Фокусник и Ассистент могут придумать стратегию, позволяющую всегда назвать слово, совпавшее хотя бы в C  битах с написанным зрителем?

Источники: Высшая проба - 2022, 11.6 (см. olymp.hse.ru)

Подсказки к задаче

Подсказка 1

Давайте начнём решать эту задачу с оценки. Причём мы будем рассуждать так, что фокусник при выписывании последовательности ошибся максимум в k символах, то есть C = 60 - k. Пусть мы знаем последовательность, написанную фокусником. Тогда с помощью какой информации на основе введённых данных мы сможем определить исходную последовательность?

Подсказка 2

Верно, нам нужно знать, в каких символах он ошибся. Тогда мы сможем однозначно определить последовательность. Но как нам это поможет в задаче? Давайте представим немного нашу задачу через множество и его покрытие. Всего возможных последовательностей, которые может написать зритель это 2^60. Давайте подумаем, как мы можем посчитать, сколько покроет одна последовательность, учитывая, что фокусник может ошибиться максимум в k знаках?

Подсказка 3

Верно, это считается с помощью числа сочетаний. Может быть такое, что он вовсе не ошибся, ошибся один раз и т.д. до максимум в k знаках. И того, для одной последовательности мы получаем сумму числа сочетаний. Но всего фокусник может получить 2^44 различных последовательностей, имея информацию от ассистента. Тогда учитывая мысль про покрытия множества размером 2^60, какую оценку мы получаем?

Подсказка 4

Да, сумма, умноженная на 2^44, должна быть больше 2^60. Но зачем? Чтобы даже, если мы ошибёмся максимум в k знаках, то мы всё равно победим, так как затронем все последовательности. Таким образом, несложной прикидкой получим оценку для k=4, откуда C=56. Теперь в следующей подсказке разберёмся с примером.

Подсказка 5

Суть нашего примера будет заключаться в том, как же действовать ассистенту, то есть какую последовательность передавать, и, что с ней делать фокуснику. Заметим, что фокусник в итоге ошибается только в 4 позициях. Давайте тогда для удобства, пока магически разобьём то, что будет передавать ассистент на блоки по 11 цифр. Тогда в каждом блоке фокусник будет что-то досчитывать, записывать и, в итоге, ошибаться максимум 1 раз в блоке. Попробуйте поугадывать, какая информация ему может понадобиться из этих 11 цифр? Стоит подумать в сторону информатики, последовательности 0 и 1 из 4 знаков и чётности, нечётности.

Подсказка 6

Давайте составим табличку из последовательностей с четырьмя символами, в которых есть хотя бы две единицы, и пронумеруем. Их будет как раз 11 штук. В последние 4 позиции блока запишем следующие "контрольные" суммы. Сначала запишем сумму по модулю два тех из первых 11 позиций, записанное 4-значное число которых имеет 1 в первом разряде. Потом аналогичное сделаем для 1 во втором разряде и т.д. до 4 разряда. Попробуем взять произвольную последовательность из 15 символов, а записать только 11. Посчитаем для них самостоятельно контрольные суммы. Теперь поизучайте эту конструкцию, например, что будет если контрольные суммы не совпадут? В скольких разрядах у них будет расхождение и где может быть ошибка – в четырех контрольных суммах или в первых 11 цифрах? А сколько в принципе может случится ошибок в такой строке, если не совпали контрольные суммы?

Подсказка 7

Ага, несложным перебором, где и сколько штук можно было допустить ошибок, понимаем, что она возможна только одна(либо в 11 цифрах, либо в контрольных суммах). Но для чего это всё таки было? Попробуйте прокрутить теперь эту конструкцию в обратном порядке. Пусть мы получили "кодовое слово" длины 15. Дальше считаем контрольные суммы по первым 11 цифрам и дописываем их. Таким образом, 15 цифр будут расходится максимум в 1 позиции. Как тогда отсюда получается окончательный пример?

Подсказка 8

Да, получается, что ассистент бьёт 60 цифр на группы по 15 и делает для них "кодовые слова" длинной 11, убирая контрольные суммы. Так и получается всего передача 44 знаков. Фокусник же далее восстанавливает контрольные суммы, как и само слово, ошибившись максимум 4 раза. Но всё таки осталось ещё понять, почему же вообще любое из слов длины 15 получается искажением какого-то из кодовых слов не более чем в одной позиции. И после этого будет победа!

Показать ответ и решение

Докажем оценку C ≤ 56.

______________________________________________________________________________________________________________________________________________________

Пусть существует стратегия, позволяющая ошибаться не более чем в k  битах (  т.е. C =60− k).  Тогда заметим, что Наблюдатель, знающий стратегию Фокусника, сообщенное Ассистентом слово, и набор бит, в которых ошибся Фокусник, может восстановить написанное Зрителем слово. В самом деле, Наблюдатель берет слово, которое должен был написать Фокусник, и меняет его в тех местах, где Фокусник ошибся. Значит, количество различных пар вида "слово сообщенное Ассистентом и набор мест, в которых фокусник ошибся"не меньше, чем различных слов, которые мог написать зритель. То есть

   (               )
244 C060+C160+ ...+ Ck60 ≥ 260

Перепишем в виде

(                )
 C060+ C160+ ...+ Ck60  ≥216

Заметим, что при k ≤3  неравенство неверно:

216 > 64000

 0    1    2   3         602  603
C60+ C60+C 60+ C60 < 1+ 60 + 2 + 6 = 1+ 60+ 1800+ 36000

_________________________________________________________________________________________________________________________________________________________________________________

Теперь попробуем показать, из каких соображений строится пример. Для начала напомним конструкцию, известную как математикам так и программистам: двоичный код длины 15, позволяющий передать 11 бит полезной информации и исправить ошибку не более чем в одном бите (также известен как 15-битный код Хэмминга). Построим его.

Для начала заметим, что есть ровно 11 слов длины 4 из нулей и единиц, содержащих хотя бы две единицы (всего слов  4
2 = 16,  минус одно из одних нулей, минус четыре с одной единицей). Припишем такие слова номерам от 1 до 11 как угодно, например как в таблице:

PIC

Теперь построим код таким образом: в первые 11 бит запишем те биты, которые хотим передать (первые 11 позиций будем называть информационными). В последние 4 бита запишем следующие контрольные суммы. В 12-й запишем сумму по модулю два тех из первых 11 бит, приписанное 4-значное число которых имеет 1 в первом разряде, то есть биты № 3,5,6,8,9,10,11. В 13-й — сумму тех из первых 11 бит, приписанное 4-значное число которых имеет 1 во втором разряде, в 14-й сумму бит, имеющих 1 в третьем разряде приписанного слова, в 15-й — в четвертом. Покажем, почему этот код позволяет исправить одну ошибку при передаче.

_________________________________________________________________________________________________________________________________________________________________________________

Пусть Получатель получил кодовое слово, возможно искаженное в одном бите. Получатель точно так же по первым 11 битам посчитает 4 контрольные суммы, и сравнит их с четырьмя полученными. Если совпали все четыре — то слово дошло без искажений. В самом деле, если бы исказился контрольный бит — в нем было бы расхождение, а если информационный — то расхождения были бы во всех контрольных, в которые он входит. Аналогично, если расхождение есть ровно в одном контрольном бите, то исказился именно он. В самом деле, если исказился другой контрольный — то все информационные дошли правильно, тогда в этом контрольном расхождения бы не было; а если исказился информационный, то все контрольные дошли правильно, и тогда расхождения были бы во всех контрольных, в которые входит искаженный информационный, а таких контрольных хотя бы два (именно за этим мы приписывали комбинации нулей и единиц, содержащие хотя бы две единицы). По аналогичным соображениям если получатель видит не менее двух расхождений с контрольными битами, то искажен точно информационный. Тогда достаточно из 11 информационных позиций выбрать ту, в приписанном 4-битном слове которой единицы стоят ровно на тех местах, на которых есть расхождения с контрольными суммами — это и есть искаженная позиция.

_________________________________________________________________________________________________________________________________________________________________________________

Теперь подумаем в других терминах, что же мы построили. У нас есть код из 211  кодовых слов. Каждое из этих слов можно исказить 16 способами (ничего не менять, или изменить один из 15 бит). Все     11
16× 2  полученных в результате слов будут разными. В самом деле, если бы какое-то слово w  получалось двумя разными способами: искажением кодового слова u1  и искажением кодового слова u2  (в обоих случаях — не более чем в одном бите), то код бы не исправлял одну ошибку — Получатель может получить слово w,  но в этом случае не может понять, посылали ему u1  или u2.  Итак, все      11
16× 2  слов разные, но это означает что вообще любое из слов длины 15 получается искажением какого-то из кодовых слов не более чем в одном бите.

На этом и построим стратегию Фокусника и Ассистента. Ассистент видит написанное зрителем 60-битное слово, режет его на четыре 15-битных слова w1,w2,w3,w4.  Как доказано выше, для них найдутся кодовые слова u1,u2,u3,u4,  такие что wi  отличается от ui  не более чем в одном символе. Тогда Ассистент выбрасывает из каждого кодового слова контрольные символы, получает четыре слова длины 11, то есть одно слово длины 44. Его он и передает Фокуснику. Фокусник восстанавливает контрольные суммы, получает слово u1u2u3u4,  отличающееся от исходного максимум в четырех битах — победа.

Ответ: 56

Ошибка.
Попробуйте повторить позже

Задача 14#74782

Через X (α)  будем обозначать точку с координатами (cosα,sinα )  (все такие лежат на окружности радиуса 1 с центром в начале координат). Выбрали произвольный угол ϕ  и провели хорды                    (   2 )
P (ϕ)P(2022ϕ),P(2022ϕ)P 2022ϕ ,...  (на шаге номер n  проводится хорда   (   n−1)      n )
P 2022  ϕ P (2022 ϕ).  Если хорда уже была проведена — она не проводится второй раз. Оказалось. что все проведенные хорды не пересекаются иначе чем по концам. Докажите, что всего проведено конечное число хорд.

Источники: Высшая проба - 2022, 11.5 (см. olymp.hse.ru)

Показать доказательство

Нам будет полезен аналог целой части < x> ,  выражающий для двух чисел с разностью x  расстояние по окружности между образами этих чисел, если намотать числовую прямую на единичную окружность: будем говорить, что <x >= {x} при      1
{x}≤ 2  и < x>= 1 − {x} при      1
{x}> 2  (здесь {x} обозначает обычную целую часть числа x  ). Тогда, например, если длина дуги между точками α  и β  равна ϕ,  то длина дуги между 2022α  и 2022β  равна < 2022ϕ> .

Для краткости точку      n
P(2022ϕ)  будем обозначать просто Pn.  Заметим, что точки не повторяются: если бы оказалось, что Pm =Pn  при m > n,  то выполнялось бы Pm+1 = Pn+1,  Pm+2 =Pn+2  и т.д., тогда число хорд было бы конечным. Итак, каждая новая точка попадает строго между ранее поставленными.

Определим по индукции понятие активной дуги n  -го шага. Для натурального n = 1  будем ей считать ту из двух дуг P0P1,  на которую попадает P2  . Заметим, что тогда все точки Pn  лежат на активной дуге первого шага. В самом деле, пусть все точки от 2 -й до m  -й лежат на активной дуге 1 -го шага, а m +1  -я там не лежит. Тогда хорды P0P1  и PmPm+1  пересекаются.

Теперь предположим, что мы уже индукцией по n  доказали, что все точки Pm  попадают на активную дугу n  -го шага при m >n.  Определим активную дугу n+ 1  -го шага. Pn+1  лежит на n  -й активной дуге, значит делит ее на две части. На одну из этих частей попадает точка Pn+2  — эту часть и будем называть активной дугой n+ 1  -го шага. Тогда чтобы индукция работала нам осталось доказать, что все точки Pm  лежат на этой дуге при m ≥n +2.  Понятно, что концы дуги — это какие-то из предыдущих точек P,  значит есть фрагмент ломанной, соединяющий их. Значит если Pm  еще лежит на дуге, а Pm+1  — уже нет, и Pm+2  не совпадает ни с одной из предыдущих точек P  (что упоминалось ранее) — значит, PmPm+1  пересекается с указанным фрагментом ломанной.

Как легко видеть, каждая следующая активная дуга является подмножеством предыдущей. Более того, обозначим через ϕn,  длину активной дуги, а через ψn  — длину дуги Pn−1Pn  (той из двух, которая лежит внутри активной). Тогда или

ϕn = ψn или  ϕn =ϕn−1− ψn
(1)

Поскольку {ϕ }∞
  n n=1  — невозрастающая последовательность положительных чисел, она имеет предел. Докажем, что этого не может быть.

Если предел равен нулю, то нулю же равен и предел последовательности     ∞
{ψn}n=1,  поскольку ψn ≤ϕn−1.  Но заметим, что ψn+1 =<2022ψn > .  То есть если     -1-
ψn ≤ 4044,  то ψn+1 =2022ψn.  Кроме того, ψn  всегда не равно нулю (иначе две точки совпали). Значит для    -1-
𝜀= 4044  в последовательности встречаются члены большие 𝜀  со сколь угодно большими номерами — ноль не является пределом.

Пусть предел равен положительному числу a.  Тогда по (1)  последовательность ψn  разбилась на две подпоследовательности, предел одной равен нулю, предел другой — a  , причем по доказанному выше вторая содержит бесконечное число членов. Заметим, что a  — неподвижная точка преобразования ψ →< 2022ψn >.  Тогда аналогично |ψn+1− a|=2022|ψn − a| если          1
|ψn− a|≤ 4044.

Выберем 𝜀< 20a22,  будем говорить о числах 0 и a  как о двух пределах. Начиная с какого-то номера все ψn  должны попадать в 𝜀  -окрестность одного из двух пределов. Но тогда при переходе от ψn  к ψn+1  расстояние до предела будет расти в 2022 раза - рано или поздно ψn  выскочит из 𝜀  -окрестности текущего предела и еще не дотянется до 𝜀  -окрестности другого предела.

Ошибка.
Попробуйте повторить позже

Задача 15#74781

Найдите все действительные числа d,  для которых существуют многочлены от одной переменной P  и Q,  такие что равенство

P(x)  P(x+ d)    1
Q(x) − Q(x+-d) = x(x-+1)

выполняется при всех значениях x  , кроме конечного числа (есть лишь конечное множество значений x  , для которых равенство не выполняется).

Источники: Высшая проба - 2022, 11.4 (см. olymp.hse.ru)

Показать ответ и решение

Сразу заметим, что при d= 0  равенство из условия невозможно, так что далее мы везде считаем, что d⁄= 0  даже когда не напоминаем об этом явно.

Предположим, что такие многочлены P  и Q  нашлись. Тогда можно считать, что они взаимнопросты (иначе поделим оба на общий множитель — новая пара тоже удовлетворяет условию), и у Q  старший коэффициент равен 1 (домножим P  и Q  на константу , чтобы старший коэффициент стал равен 1). Введем обозначение для разложения Q  на линейные множители (естественно, воспользовавшись существованием такого разложения в комплексных числах):

Q(x)=(x− α1)n1(x− α2)n2⋅⋅⋅(x− αk)nk

Для комплексного числа α  множество чисел вида α +md,  где m ∈ℤ  , будем называть цепью числа α.

______________________________________________________________________________________________________________________________________________________

Ключевое утверждение:

Если α  — корень Q,  то числа 0 и 1 принадлежат цепи α.

______________________________________________________________________________________________________________________________________________________

Доказательство.

Пусть α  — корень Q,  тогда обозначим через m− и m+  такие минимальное и максимальное значения m,  при которых α+ md  является корнем Q.  Заметим, что m− и m+  определены корректно: множество значений m  не пусто (поскольку 0 подходит) и конечно, поскольку у Q  конечное число корней (первое место, в котором важно, что d⁄= 0  ). Тогда пусть не оба числа 0 и 1 лежат в цепи α.  Тогда одно из двух чисел α+ (m− − 1)d  и α+ m+d  не является ни 0 ни 1 (второе место: нам важно, что α +(m − − 1)d  и α +m+d  — два разных числа). Рассмотрим эти два случая.

Пусть α+ m+d= αi  — не равно ни 0 ни 1. Посмотрим на равенство из условия

P-(x)− P(x+-d)= ---1-- = 1− -1--
Q (x)  Q(x+ d)  x(x+ 1)   x  x+ 1

и разложим левую часть на простейшие дроби:

P(x)= P (x)+ --P1(x)-- +--P2(x)- +⋅⋅⋅+--Pk(x)- ,
Q (x)   0    (x− α1)n1  (x− α2)n2      (x− αk)nk

где степень Pi(x)  меньше ni  при 1 ≤i≤ k,  причем Pi ⁄= 0.

Поскольку αi  — корень Q,  в разложение PQ((xx))  входит член со знаменателем (x − αi)ni  и ненулевым числителем. Но αi  — не корень Q (x+ d),  иначе α + d= α+ (m  +1)d
 i          +  было бы корнем Q(x),  что противоречило бы максимальности m
 +  . Тогда член со знаменателем      ni
(x− αi) не входит в разложение P(x+d)
Q(x+d),  значит члену с таким знаменателем слева не с чем сократиться — но он не входит в правую часть — противоречие.

_________________________________________________________________________________________________________________________________________________________________________________

Итак, мы доказали, что если у многочлена Q  есть комплексные корни, то в цепь этого корня входят числа 0 и 1, то есть выполняется равенство    -1
d= m  для какого-то целого m  . Если же у Q  нет комплексных корней, то он - ненулевая константа, то есть P(x)-
Q(x)  и P (x+d)
Q-(x+d) — многочлены, тогда их разность не может равняться x(x1+1).

Осталось показать, что все значения вида d= 1m,  где m ∈ ℤ,m ⁄= 0  подходят. Для m >0  достаточно взять функцию

1+ --11-+ --12-+ ⋅⋅⋅+ ---1m−1-
x  x+ m   x+ m       x+  m

и привести сумму к общему знаменателю, числитель взять в качестве P,  а знаменатель — Q.  Для m <0  то же самое сделать с суммой

-−1-  ---−1---  --−-1---      -−-1--
x+ 1 + x+ −m−−m1-+ x+ −m−−m2-+⋅⋅⋅+ x+ −1m
Ответ:

 d =-1,
   m  где m ∈ℤ,m ⁄= 0

Ошибка.
Попробуйте повторить позже

Задача 16#74780

Гипотенуза AB  прямоугольного треугольника ABC  касается вписанной и соответствующей вневписанной окружностей в точках T ,T
 1 2  соответственно. Окружность, проходящая через середины сторон, касается этих же окружностей в точках S1,S2  соответственно. Докажите, что ∠S1CT1 = ∠S2CT2.

Источники: Высшая проба - 2022, 11.3 (см. olymp.hse.ru)

Подсказки к задаче

Подсказка 1

Понятно, что скорее всего стандартный счёт углов тут не поможет. Здесь у нас и окружность Эйлера, и вписанная, и вневписанная. Так что либо нужны большие знания геометрических конструкций, либо какие-то хитрости. Пойдём хитрым путём. У нас есть как минимум три окружности на картинке, причём какие-то касающиеся. Какое преобразование плоскости тогда напрашивается сделать?

Подсказка 2

Верно, давайте сделаем инверсию с центром в точке C, причём сделать её с произвольным радиусом не слишком удобно. Давайте сделаем инверсию с радиусом √(ab/2), где переменные это стандартное обозначение сторон. Но если вы начнёте рисовать новую картинку, выйдет что-то не слишком хорошее. Какое ещё преобразование плоскости хорошо будет применить?

Подсказка 3

Смотрите, а давайте после инверсии сделаем ещё симметрию. Вот теперь осталось только понять, что и куда переходит после преобразований окончательно и почему мы выбрали такой радиус инверсии(на самом деле можно было без него, но так удобнее). В конце концов мы поймём, что углы переходят в друг друга, и победа!

Показать доказательство

Введём обозначения для длин сторон: BC = a,AC = b,AB = c.

PIC

Сделаем инверсию с центром C  и радиусом    ∘ ---
R=   12ab  с симметрией относительно биссектрисы угла C.

Середины сторон прямоугольного треугольника и вершина его прямого угла образуют прямоугольник, значит, все четыре на одной окружности. Значит, при инверсии образ окружности — прямая. Легко посчитать, что эта прямая отсекает от лучей CA  и CB  отрезки длины a  и b  соответственно, то есть симметрична AB  относительно биссектрисы угла A.  Поэтому гипотенуза и окружность Эйлера треугольника переходят друг в друга.

Касательная из C  к вписанной окружности равна её радиусу r,  а касательная из C  к вневписанной окружности равна полупериметру p.  Таким образом, их произведение pr= S(ABC)  — площади треугольника ABC.  Итак, pr= R2.  Поэтому вписанная и вневписанная окружности треугольника ABC  переходят друг в друга.

Следовательно, T1  переходит в S2,  а T2  переходит в S1.  Угол ∠S1CT1  переходит в угол ∠S2CT2,  значит, они равны.

Ошибка.
Попробуйте повторить позже

Задача 17#74779

Вася пришел в казино, имея один вшэ-коин (единственную в мире виртуальную валюту, которую можно делить на любые части; например, можно поставить на кон π-
10  вшэ-коина). В казино игрокам предлагается делать ставки на цвет шара, который будет вытащен из ящика. Фиксировано число p,  причем 1 <p <2.  Если цвет вытащенного шара совпадает с тем, на который игрок поставил x  денег — игрок получит назад px  денег, если не совпадает — не получит ничего. Для ставок в каждом раунде можно использовать не только деньги, имевшиеся к началу игры, но и выигрыши прошлых раундов. Перед началом игры Вася смог подсмотреть, что в ящик положили 3 черных и 3 красных шара (других шаров нет), сыгранные шары обратно в ящик не возвращаются, игра происходит пока ящик не опустеет. Какую максимальную сумму Вася может гарантированно иметь к концу розыгрыша?

Источники: Высшая проба - 2022, 11.2 (см. olymp.hse.ru)

Подсказки к задаче

Подсказка 1

Мы понимаем, что у нас в задаче Вася может максимум 6 раз сделать ставку. Тогда имеет смысл "перебрать случаи". Давайте будем заполнять табличку 4 на 4(0, 1, 2 и 3 красных шара по вертикали и аналогично для чёрных по горизонтали), которая будет говорить о том, во сколько раз увеличится выигрыш, когда осталось определённое число шаров. Тогда во сколько раз увеличатся деньги, после того как у нас останутся шары только одного цвета?

Подсказка 2

Верно, они могут увеличиваться в p, p^2, p^3 раз. Вася будет просто ставить всю сумму каждый раз на один цвет. Это то, что будет стоять в крайних столбцах и диагоналях. Ещё понятно, если шаров нет, то мы ничего не выигрываем, то есть 1 коэффициент в клетке (0; 0). Давайте поймём такую вещь, а есть ли Васе смысл ставить деньги на оба шара?

Подсказка 3

Да, смысла в этом нет, Васе надо ставить какую-то часть денег только на один цвет. Если он поставит a денег на один цвет, а b на другой, то получит на 2(p-b) денег меньше, чем если бы поставил a-b на один цвет. Это несложно посчитать. Теперь давайте попробуем разобраться с другими клетками. Например, если остался 1 чёрный и 1 красный шарик. Во сколько раз мы получим больше денег точно? Полезно будет ввести неизвестные.

Подсказка 4

Верно, мы получим больше в p раз. Дело в том, что даже если мы не угадаем шар, то следующим ходом увеличим в p раз количество денег. Давайте теперь попробуем в общем виде провести рассуждения. Если у Васи было X денег, а поставил он aX, где a какое-то число от 0 до 1. Значит, мы можем посчитать случаи, когда Вася угадал и когда не угадал. Но тогда сколько гарантированно он получит? Что это может значить с точки зрения полученных формул?

Подсказка 5

Да, гарантированный выигрыш — это минимум из двух наших выражений. Но можно заметить, что одно из них убывающее, а другое возрастающее от a. Значит, и минимум будет, когда они равны. Отсюда мы получаем a, а потом и во сколько раз увеличится наше количество денег. Так мы, постепенно заполняя таблицу, получим, что должно быть в клетке, где 3 шара каждого цвета. Это и будет ответ на задачу, победа!

Показать ответ и решение

Заполним табличку: в клетке (i,j)  запишем, на какое максимальное число Вася может гарантированно к концу игры умножить имеющуюся у него сейчас сумму, если сейчас в ящике осталось i  черных и j  красных шаров. Легко понять, что стоит с краю: если уже не осталось черных шаров, то Вася может смело ставить все деньги на красный шар, соответственно увеличивая капитал в p  раз за каждый из оставшихся красных шаров. Аналогично если не осталось красных. Это и отмечено в таблице ниже.

PIC

Теперь поймем, что должно стоять в клетке (i,j)  если мы уже знаем, что в клетках (i− 1,j)  и (i,j− 1)  стоят числа x  и y  соответственно. Пусть для определенности x≤ y.

Во-первых, в оптимальной стратегии Вася не должен делать положительные ставки на оба исхода. В самом деле, пусть по своей стратегии он должен сейчас поставить суммы a  и b,  причем a ≥b >0.  Тогда пусть вместо этого он поставит a− b  денег на тот исход, на который должен был ставить a,  и на 2b  больше денег оставит не поставленными. Тогда при любом исходе он будет иметь на (2− p)b  денег больше, чем имел бы, если бы ставил a  и b.

Теперь поймём, сколько же Вася должен ставить. Ставить он должен на тот цвет, выпадание которого приводит в клетку с числом   x  (  напомним, x ≤y),  в противном случае если этот цвет выпадет, Вася не сможет увеличить свой капитал более чем в x  раз, а мы строим стратегию лучше. Для определенности обозначим количество Васиных денег через D  и пусть он поставит 𝜀D  денег на цвет, выпадение которого приводит в клетку с числом x.  Тогда если выпал этот цвет Вася оказался в этой клетке имея (1+ (p− 1)𝜀)D  денег, соответственно закончит игру, имея не менее (1+ (p− 1)𝜀)xD  денег (и не может гарантированно иметь больше). Если же выпал цвет, приводящий в клетку с числом y,  Вася попал туда, имея (1− 𝜀)D  денег, значит закончит игру, имея не меньше (1− 𝜀)yD  денег (и не может гарантированно иметь больше). Итак, гарантированный минимум при этой стратегии есть min((1+ (p − 1)𝜀)xD,(1 − 𝜀)yD ).  Поскольку первая из функций под минимумом возрастающая по 𝜀  , а вторая — убывающая, максимум минимума достигается при значении 𝜀,  для которого функции принимают одно значение. Имеем

(1+(p− 1)𝜀)xD = (1− 𝜀)yD

𝜀 = --y−-x---
    y+(p− 1)x

То есть

(1+ (p − 1)𝜀)xD = (1− 𝜀)yD )=---pxy---D
                         y+(p− 1)x

Иными словами, в интересующей нас клетке должно стоять число

---pxy---
y+ (p− 1)x

Пользуясь этой формулой и значениями в клетках на краях, заполним всю табличку:

PIC

Ответ:

----p5---
5p2− 6p+2

Ошибка.
Попробуйте повторить позже

Задача 18#74778

В этой задаче запись x modn,  где x  — целое а n  — натуральное, обозначает такое целое число y  от 0 до n− 1,  что x− y  делится на n.

Существует ли такая функция f,  определенная для целых значений аргумента и принимающая целые значения, что при любом целом x  верно

 (( 2  )     )  (   2  )
f  x +1 mod 7 = f(x) +1 mod 11 ?

Источники: Высшая проба - 2022, 11.1 (см. olymp.hse.ru)

Подсказки к задаче

Подсказка 1

Такс, перед нами функциональное уравнение, да еще и аргумент функции мы берем по модулю 7… Давайте вспомним, что мы обычно делаем в функциональных уравнениях?

Подсказка 2

Верно, подставляем хорошие значения! А какие значения хочется подставить в это уравнение(не забывайте, что в левой части аргумент берется по модулю)

Подсказка 3

Да, хочется найти такие x, для которых верно: x = x²+1 по модулю 7. Почему так хочется сделать? Если получится найти такой x, то дальше уравнение сведется к f(x) = (f(x)²+1) (по модулю 11). А понять, решается ли такое уравнение уже проще, чем решить исходное! Остаётся найти такие x.

Подсказка 4

Заметим, что 3 = 3²+1 по модулю 7! То есть, 3 нам подходит. Что можно сказать про f(3)?

Подсказка 5

Верно, f(3) = f(3)²+1 по модулю 11. Мы получили почти то же самое, что и на одном из предыдущих шагов, только теперь по модулю 11! Остаётся показать, что таких y не существует.

Показать ответ и решение

Стандартным ходом при решении задач на функциональные уравнения является подставить какое-то значение переменной, при котором два часто возникающих и не равных друг-другу тождественно выражения оказываются равны, и посмотреть, какие следствия из этого удастся вывести. Применительно к данной задаче на роль такой подстановки простится значение x0,  для которого выполнялось бы      2
x0 = x0+ 1mod 7.

Задумаемся, а существует ли такое x0?  Условие равносильно квадратному уравнению в остатках(в этом абзаце все сравнимости по модулю 7):

x20− x0 +1 ≡0

    1±√ −3-  1±√ −3+-7  {3 − 1}  {3 +7 −1 +7}
x0 ≡--2----≡ ----2----≡  2,-2- ≡  --2-,--2--  ≡ {5,3}

Или можно было просто перебором остатков, благо их всего 7, убедиться, что любой из 3 и 5 подходят.

Что же нам дает равенство 3 =32+ 1mod 7?  Просится от обоих частей взять функцию f,  а затем воспользоваться условием задачи. Имеем:

f(3)=f ((32+ 1)mod7)= (f(3)2+ 1) mod11

Чтобы подчеркнуть полученное, обозначим f(3)= y  и выбросим среднюю часть:

   ( 2  )
y = y + 1 mod11

Отсюда следует (далее все сравнимости будут по модулю 11)

y2− y+ 1≡ 0

Отметим что это именно следствие, а не равносильность. Выясним, имеет ли сравнимость решения, действуя стандартно      √ --
y ≡ 1±-2−3.  А извлекается ли квадратный корень из -3 по модулю 11? Заметим что 12 ≡ (−1)2 ≡1,22 ≡(−2)2 ≡ 4,32 ≡ (− 3)2 ≡ 9,  42 ≡ (− 4)2 ≡ 5  и 52 ≡ (−5)2 ≡3.  Мы перебрали все остатки, среди квадратов не нашлось -3, значит корень не извлекается, значит уравнение y2− y+ 1≡ 0  не имеет решений.

Итак, требуемой функции f  не существует.

Ответ: нет

Ошибка.
Попробуйте повторить позже

Задача 19#75303

Для таблички n ×n  рассматриваем семейство квадратов 2× 2,  состоящих из клеток таблицы, и обладающее свойством: для любого квадрата семейства найдется покрытая им клетка, не покрытая никаким другим квадратом из семейства. Через f(n)  обозначим максимальное количество квадратов в таком семействе. Для какого наименьшего C  неравенство         2
f(n)≤ Cn  верно при любом n?

Показать ответ и решение

Во-первых, докажем что f(n)≤ 1n2.
      2  Для этого полезно доказать более сильное утверждение: для произвольной фигуры из S  клеток количество квадратов 2× 2  в семействе, таком что все квадраты лежат в фигуре и для любого квадрата найдется клетка, покрытая только им, не превосходит S∕2.  Рассмотрим два случая: для семейства найдется клетка A,  покрытая четырьмя квадратами, и случай, когда такой клетки не найдется.

Если такая клетка A  нашлась, то рассмотрим четыре покрывающих ее квадрата. Они образуют квадрат 3× 3  с клеткой A  в центре. И поскольку в каждом из четырех квадратов 2×2  должна быть клетка, покрытая только им — это четыре угловые клетки квадрата 3× 3,  поскольку все остальные покрыты хотя бы дважды. Но тогда никакой другой квадрат 2×2  из семейства не покрывает клетки квадрат 3 ×3,  иначе он покрывает и угловую клетку, а она должна быть покрыта только один раз. Итак, все остальные квадраты лежат в множестве площади S − 9.  В этот момент доказательства еще не поздно решить, что на самом-то деле мы ведем индукцию по S  :), благо база тривиальна. Итак, всего квадратов в семействе оказывается не больше 4+ S−29 = S−21< S∕2.

Пусть клетки, покрытой четырьмя квадратами из семейства, не найдется. Поместим в каждую клетку множества единичный заряд. Теперь пусть каждая клетка, покрытая k  квадратами из семейства, отдаст каждому из этих квадратов по 1∕k  заряда (таким образом, раздаст весь свой заряд). Тогда каждый квадрат семейства получил заряд не меньше 2,  потому что минимум от одной клетки получил 1,  и от остальных получал не меньше 1∕3  от каждой. Итого, всего полученного заряда не меньше чем дважды число квадратов в семействе, а отданного заряда не больше S,  итого квадратов в семействе не больше S∕2.

Теперь построим пример, доказывающий, что f(n)≥ 12n2− 4n,  следовательно неравенство f(n) ≤Cn2  при C < 12  неверно при всех достаточно больших n.

Возьмем бесконечную клетчатую плоскость и покрасим ее в два цвета следующим образом: выберем одно из двух направлений диагонали, покрасим все клетки каждой диагонали в один цвет: две диагонали в белый, следующие две в черный и так далее с периодом 4.  Теперь выберем квадрат n ×n,  в который черных клеток попало не меньше чем белых, то есть хотя бы n2∕2.  Теперь на каждую черную клетку внутри квадрата n ×n  положим квадрат 2 ×2  так, чтобы кроме этой черной клетки квадрат содержал только белые (это можно сделать единственным образом). Удалим все квадраты 2×2,  частично вылезшие за границы квадрата n ×n,  их не больше 4n.  Требуемое семейство построено.

Ответ:

 C = 1
    2

Ошибка.
Попробуйте повторить позже

Задача 20#89083

В последовательности чисел Фибоначчи 1,1,2,3,5,8,13,21,...  каждое следующее число, начиная с третьего, равно сумме двух предыдущих. Докажите, что среди чисел Фибоначчи нет ни одной натуральной степени числа 7.

Подсказки к задаче

Подсказка 1

Какое первое число в последовательности чисел Фибоначчи кратно 7? Чему равен его индекс?

Подсказка 2

Первое число Фибоначчи, кратное 7 - это 21, которое является 8 числом Фибоначчи. Продолжив выписывать элементы данной последовательности можно заметить, что первые несколько членов, индексы которых кратны 8, делятся на 7. Докажите, что на 7 делятся те и только те члены последовательности чисел Фибоначчи, индексы которых кратны 8. Как можно доказать, что никакое из данных чисел не является степенью 7?

Подсказка 3

Показать, что каждое из них имеет простой делитель отличный от 7. Можно ли это сделать, найдя зависимость делимости членов последовательности от их индекса, аналогичную уже полученной?

Подсказка 4

Да, достаточно показать, что на 3 делятся те и только те числа Фибоначчи, номер которых делится на 4.

Показать доказательство

Для начала докажем, что на 7  делятся те и только те числа Фибоначчи, номер которых делится на 8.  Докажем это по индукции. База: Первое число Фибоначчи, кратное 7  — это 21,  которое является 8  числом Фибоначчи.

Переход: Пусть этот факт был верен для всех чисел Фибоначчи с номерами от 1  до 8k.  Докажем, что он верен для чисел от 8k+1  до 8k+ 8.  Пусть число с номером 8k − 1  имело остаток a  от деления на 7(a ⁄=0).  Тогда числа с номерами 8k +1,...,8k +8  будут иметь следующие остатки: a,a,2a,3a,5a,a,6a,0.

Теперь докажем, что на 3  делятся те и только те числа Фибоначчи, номер которых делится на 4.  Доказательство аналогично.

Следовательно, если число Фибоначчи делится на 7,  то его номер делится на 8.  Значит его номер делится на 4,  а значит, само число обязано делиться на 3.  Значит оно не может быть равно натуральной степени числа 7.

Рулетка
Вы можете получить скидку в рулетке!